LSAT and Law School Admissions Forum

Get expert LSAT preparation and law school admissions advice from PowerScore Test Preparation.

User avatar
 Dave Killoran
PowerScore Staff
  • PowerScore Staff
  • Posts: 5852
  • Joined: Mar 25, 2011
|
#94640
Complete Question Explanation
(The complete setup for this game can be found here: lsat/viewtopic.php?f=296&t=2115)

The correct answer choice is (A)

The question stem in this problem is a bit odd, and most people read it at least twice before getting a clear picture of what is being asked. The question asks for who could leave the first and sixth messages, and have those two messages be their only two messages. So, a caller such as H is immediately eliminated because the third rule stipulates that if H makes the first call, P makes the last call. This removes answer choice (B) from consideration.

The fact that a caller leaves exactly two messages means that the 2-1-1-1-1-0 distribution is in effect:

G2-Q7-d1.png

Accordingly, L and T can also be eliminated as they can never leave the first message, removing answer choices (C) and (E) from consideration. P can also be eliminated as P > T, and therefore P can never leave the last message. This removes answer choice (D) from consideration.

Hence, answer choice (A) is correct.
You do not have the required permissions to view the files attached to this post.
 ronnieronnie
  • Posts: 16
  • Joined: Jan 24, 2012
|
#3465
So I was working through this game and I got number 7 wrong but I don't understand the question because I cannot figure out why A is the correct answer. I thought the question was asking who can have the first, second, and last messages on the answering machine. F cannot have all three of these because it leaves only three spaces for the four variables that must accompany choosing F. So could you guys please explain what exactly the question is asking so that I can avoid this mistake in the future?

Thanks,
Ronnie
User avatar
 Dave Killoran
PowerScore Staff
  • PowerScore Staff
  • Posts: 5852
  • Joined: Mar 25, 2011
|
#3473
Hi Ronnie,

I agree that the question stem in this problem is a bit odd, and most people read it at least twice before getting some idea of what is being asked. Let's see if we can figure out what's going on here.

The question asks for who could leave the first and sixth messages, and have those two messages be their only two messages. So, someone out there leaves messages 1 and 6, and that's their entire message profile. With a caller leaving exactly two messages, we can infer that a 2-1-1-1-1-0 distribution of messages-to-people is in effect: 2-1-1-1-1-0.

Under the 2-1-1-1-1-0 distribution, F, P, T, H, and L each leave a message, with P > T and H > L.

H cannot be first because the third rule stipulates that if H makes the first call, P makes the last call. This removes answer choice (B) from consideration.

Accordingly, L and T can also be eliminated as they can never leave the first message, removing answer choices (C) and (E) from consideration. P can also be eliminated as P > T, and therefore P can never leave the last message. This removes answer choice (D) from consideration.

Hence, answer choice (A) is correct.

Please let me know if that helps. Thanks!
User avatar
 German.Steel
  • Posts: 55
  • Joined: Jun 12, 2021
|
#99318
FWIW: I'm a recent 99.5th percentile scorer, and I had NO IDEA what this question was asking, lol. Thank goodness they don't ask convoluted nonsense like this nowadays!

Get the most out of your LSAT Prep Plus subscription.

Analyze and track your performance with our Testing and Analytics Package.